converge pointwise but not uniformly











up vote
1
down vote

favorite












How can I prove that
$$sum_{n=1}^infty frac{sin(nx)}{sqrt{n}}$$
converges pointwise on $[-pi, pi]$ but not uniformly?



For the pointwise part, I tried to prove it by comparison, using
$$sum_{n=1}^infty frac{sin(nx)}{sqrt{n}} leq sum_{n=1}^infty frac{nx}{sqrt{n}} = sum_{n=1}^infty xsqrt{n},$$
which does not converge.



I also tried
$$ sum_{n=1}^infty left| frac{sin(nx)}{sqrt{n}}right| leq sum_{n=1}^infty left| frac{1}{sqrt{n}}right|,$$
which does not converge either.










share|cite|improve this question




























    up vote
    1
    down vote

    favorite












    How can I prove that
    $$sum_{n=1}^infty frac{sin(nx)}{sqrt{n}}$$
    converges pointwise on $[-pi, pi]$ but not uniformly?



    For the pointwise part, I tried to prove it by comparison, using
    $$sum_{n=1}^infty frac{sin(nx)}{sqrt{n}} leq sum_{n=1}^infty frac{nx}{sqrt{n}} = sum_{n=1}^infty xsqrt{n},$$
    which does not converge.



    I also tried
    $$ sum_{n=1}^infty left| frac{sin(nx)}{sqrt{n}}right| leq sum_{n=1}^infty left| frac{1}{sqrt{n}}right|,$$
    which does not converge either.










    share|cite|improve this question


























      up vote
      1
      down vote

      favorite









      up vote
      1
      down vote

      favorite











      How can I prove that
      $$sum_{n=1}^infty frac{sin(nx)}{sqrt{n}}$$
      converges pointwise on $[-pi, pi]$ but not uniformly?



      For the pointwise part, I tried to prove it by comparison, using
      $$sum_{n=1}^infty frac{sin(nx)}{sqrt{n}} leq sum_{n=1}^infty frac{nx}{sqrt{n}} = sum_{n=1}^infty xsqrt{n},$$
      which does not converge.



      I also tried
      $$ sum_{n=1}^infty left| frac{sin(nx)}{sqrt{n}}right| leq sum_{n=1}^infty left| frac{1}{sqrt{n}}right|,$$
      which does not converge either.










      share|cite|improve this question















      How can I prove that
      $$sum_{n=1}^infty frac{sin(nx)}{sqrt{n}}$$
      converges pointwise on $[-pi, pi]$ but not uniformly?



      For the pointwise part, I tried to prove it by comparison, using
      $$sum_{n=1}^infty frac{sin(nx)}{sqrt{n}} leq sum_{n=1}^infty frac{nx}{sqrt{n}} = sum_{n=1}^infty xsqrt{n},$$
      which does not converge.



      I also tried
      $$ sum_{n=1}^infty left| frac{sin(nx)}{sqrt{n}}right| leq sum_{n=1}^infty left| frac{1}{sqrt{n}}right|,$$
      which does not converge either.







      sequences-and-series convergence uniform-convergence pointwise-convergence






      share|cite|improve this question















      share|cite|improve this question













      share|cite|improve this question




      share|cite|improve this question








      edited Nov 21 at 21:40









      rafa11111

      1,025417




      1,025417










      asked Nov 21 at 21:04









      kit kat

      91




      91






















          2 Answers
          2






          active

          oldest

          votes

















          up vote
          2
          down vote













          For the pointwise convergence, use Dirichlet's test.



          To see that it's not uniform, note that $sin(nx) > 2 n x/pi$ if $0 < n x < pi/2$. Take $x = 1/N$ and consider the $N$'th partial sum



          $$ eqalign{sum_{n=1}^N frac{sin(n/N)}{
          sqrt{n}} &ge frac{2}{pi N} sum_{n=1}^N sqrt{n} cr&ge frac{2}{pi N} int_0^N
          sqrt{t}; dt = frac{4sqrt{N}}{3pi}}$$






          share|cite|improve this answer




























            up vote
            1
            down vote













            Pointwise convergence is granted by Dirichlet's test, but a series of continuous functions cannot converge uniformly to an unbounded function. We may compute $lim_{xto 0^+}sum_{ngeq 1}frac{sin(n x)}{sqrt{n}}$ through a convolution with an approximate identity:



            $$begin{eqnarray*}lim_{xto 0^+}sum_{ngeq 1}frac{sin(n x)}{sqrt{n}}&=&lim_{mto +infty}sum_{ngeq 1}frac{1}{sqrt{n}}int_{0}^{+infty}msin(nx) e^{-mx},dx\&=&lim_{mto +infty}sum_{ngeq 1}frac{1}{sqrt{n}}cdotfrac{mn}{m^2+n^2}.end{eqnarray*}$$
            The RHS is $+infty$ since by Riemann sums
            $$ lim_{mto +infty}sum_{ngeq 1}frac{sqrt{mn}}{m^2+n^2}=int_{0}^{+infty}frac{sqrt{x}}{1+x^2},dx = frac{pi}{sqrt{2}}.$$
            Much simpler, once $f(x)$ is defined as $sum_{ngeq 1}frac{sin(nx)}{sqrt{n}}$ we have
            $$ int_{-pi}^{pi}f(x)^2,dx = pisum_{ngeq 1}frac{1}{n} = +infty $$
            by Parseval's theorem, hence $f(x)$ cannot be bounded on $[-pi,pi]$.






            share|cite|improve this answer























              Your Answer





              StackExchange.ifUsing("editor", function () {
              return StackExchange.using("mathjaxEditing", function () {
              StackExchange.MarkdownEditor.creationCallbacks.add(function (editor, postfix) {
              StackExchange.mathjaxEditing.prepareWmdForMathJax(editor, postfix, [["$", "$"], ["\\(","\\)"]]);
              });
              });
              }, "mathjax-editing");

              StackExchange.ready(function() {
              var channelOptions = {
              tags: "".split(" "),
              id: "69"
              };
              initTagRenderer("".split(" "), "".split(" "), channelOptions);

              StackExchange.using("externalEditor", function() {
              // Have to fire editor after snippets, if snippets enabled
              if (StackExchange.settings.snippets.snippetsEnabled) {
              StackExchange.using("snippets", function() {
              createEditor();
              });
              }
              else {
              createEditor();
              }
              });

              function createEditor() {
              StackExchange.prepareEditor({
              heartbeatType: 'answer',
              convertImagesToLinks: true,
              noModals: true,
              showLowRepImageUploadWarning: true,
              reputationToPostImages: 10,
              bindNavPrevention: true,
              postfix: "",
              imageUploader: {
              brandingHtml: "Powered by u003ca class="icon-imgur-white" href="https://imgur.com/"u003eu003c/au003e",
              contentPolicyHtml: "User contributions licensed under u003ca href="https://creativecommons.org/licenses/by-sa/3.0/"u003ecc by-sa 3.0 with attribution requiredu003c/au003e u003ca href="https://stackoverflow.com/legal/content-policy"u003e(content policy)u003c/au003e",
              allowUrls: true
              },
              noCode: true, onDemand: true,
              discardSelector: ".discard-answer"
              ,immediatelyShowMarkdownHelp:true
              });


              }
              });














              draft saved

              draft discarded


















              StackExchange.ready(
              function () {
              StackExchange.openid.initPostLogin('.new-post-login', 'https%3a%2f%2fmath.stackexchange.com%2fquestions%2f3008359%2fconverge-pointwise-but-not-uniformly%23new-answer', 'question_page');
              }
              );

              Post as a guest















              Required, but never shown

























              2 Answers
              2






              active

              oldest

              votes








              2 Answers
              2






              active

              oldest

              votes









              active

              oldest

              votes






              active

              oldest

              votes








              up vote
              2
              down vote













              For the pointwise convergence, use Dirichlet's test.



              To see that it's not uniform, note that $sin(nx) > 2 n x/pi$ if $0 < n x < pi/2$. Take $x = 1/N$ and consider the $N$'th partial sum



              $$ eqalign{sum_{n=1}^N frac{sin(n/N)}{
              sqrt{n}} &ge frac{2}{pi N} sum_{n=1}^N sqrt{n} cr&ge frac{2}{pi N} int_0^N
              sqrt{t}; dt = frac{4sqrt{N}}{3pi}}$$






              share|cite|improve this answer

























                up vote
                2
                down vote













                For the pointwise convergence, use Dirichlet's test.



                To see that it's not uniform, note that $sin(nx) > 2 n x/pi$ if $0 < n x < pi/2$. Take $x = 1/N$ and consider the $N$'th partial sum



                $$ eqalign{sum_{n=1}^N frac{sin(n/N)}{
                sqrt{n}} &ge frac{2}{pi N} sum_{n=1}^N sqrt{n} cr&ge frac{2}{pi N} int_0^N
                sqrt{t}; dt = frac{4sqrt{N}}{3pi}}$$






                share|cite|improve this answer























                  up vote
                  2
                  down vote










                  up vote
                  2
                  down vote









                  For the pointwise convergence, use Dirichlet's test.



                  To see that it's not uniform, note that $sin(nx) > 2 n x/pi$ if $0 < n x < pi/2$. Take $x = 1/N$ and consider the $N$'th partial sum



                  $$ eqalign{sum_{n=1}^N frac{sin(n/N)}{
                  sqrt{n}} &ge frac{2}{pi N} sum_{n=1}^N sqrt{n} cr&ge frac{2}{pi N} int_0^N
                  sqrt{t}; dt = frac{4sqrt{N}}{3pi}}$$






                  share|cite|improve this answer












                  For the pointwise convergence, use Dirichlet's test.



                  To see that it's not uniform, note that $sin(nx) > 2 n x/pi$ if $0 < n x < pi/2$. Take $x = 1/N$ and consider the $N$'th partial sum



                  $$ eqalign{sum_{n=1}^N frac{sin(n/N)}{
                  sqrt{n}} &ge frac{2}{pi N} sum_{n=1}^N sqrt{n} cr&ge frac{2}{pi N} int_0^N
                  sqrt{t}; dt = frac{4sqrt{N}}{3pi}}$$







                  share|cite|improve this answer












                  share|cite|improve this answer



                  share|cite|improve this answer










                  answered Nov 21 at 21:20









                  Robert Israel

                  315k23206456




                  315k23206456






















                      up vote
                      1
                      down vote













                      Pointwise convergence is granted by Dirichlet's test, but a series of continuous functions cannot converge uniformly to an unbounded function. We may compute $lim_{xto 0^+}sum_{ngeq 1}frac{sin(n x)}{sqrt{n}}$ through a convolution with an approximate identity:



                      $$begin{eqnarray*}lim_{xto 0^+}sum_{ngeq 1}frac{sin(n x)}{sqrt{n}}&=&lim_{mto +infty}sum_{ngeq 1}frac{1}{sqrt{n}}int_{0}^{+infty}msin(nx) e^{-mx},dx\&=&lim_{mto +infty}sum_{ngeq 1}frac{1}{sqrt{n}}cdotfrac{mn}{m^2+n^2}.end{eqnarray*}$$
                      The RHS is $+infty$ since by Riemann sums
                      $$ lim_{mto +infty}sum_{ngeq 1}frac{sqrt{mn}}{m^2+n^2}=int_{0}^{+infty}frac{sqrt{x}}{1+x^2},dx = frac{pi}{sqrt{2}}.$$
                      Much simpler, once $f(x)$ is defined as $sum_{ngeq 1}frac{sin(nx)}{sqrt{n}}$ we have
                      $$ int_{-pi}^{pi}f(x)^2,dx = pisum_{ngeq 1}frac{1}{n} = +infty $$
                      by Parseval's theorem, hence $f(x)$ cannot be bounded on $[-pi,pi]$.






                      share|cite|improve this answer



























                        up vote
                        1
                        down vote













                        Pointwise convergence is granted by Dirichlet's test, but a series of continuous functions cannot converge uniformly to an unbounded function. We may compute $lim_{xto 0^+}sum_{ngeq 1}frac{sin(n x)}{sqrt{n}}$ through a convolution with an approximate identity:



                        $$begin{eqnarray*}lim_{xto 0^+}sum_{ngeq 1}frac{sin(n x)}{sqrt{n}}&=&lim_{mto +infty}sum_{ngeq 1}frac{1}{sqrt{n}}int_{0}^{+infty}msin(nx) e^{-mx},dx\&=&lim_{mto +infty}sum_{ngeq 1}frac{1}{sqrt{n}}cdotfrac{mn}{m^2+n^2}.end{eqnarray*}$$
                        The RHS is $+infty$ since by Riemann sums
                        $$ lim_{mto +infty}sum_{ngeq 1}frac{sqrt{mn}}{m^2+n^2}=int_{0}^{+infty}frac{sqrt{x}}{1+x^2},dx = frac{pi}{sqrt{2}}.$$
                        Much simpler, once $f(x)$ is defined as $sum_{ngeq 1}frac{sin(nx)}{sqrt{n}}$ we have
                        $$ int_{-pi}^{pi}f(x)^2,dx = pisum_{ngeq 1}frac{1}{n} = +infty $$
                        by Parseval's theorem, hence $f(x)$ cannot be bounded on $[-pi,pi]$.






                        share|cite|improve this answer

























                          up vote
                          1
                          down vote










                          up vote
                          1
                          down vote









                          Pointwise convergence is granted by Dirichlet's test, but a series of continuous functions cannot converge uniformly to an unbounded function. We may compute $lim_{xto 0^+}sum_{ngeq 1}frac{sin(n x)}{sqrt{n}}$ through a convolution with an approximate identity:



                          $$begin{eqnarray*}lim_{xto 0^+}sum_{ngeq 1}frac{sin(n x)}{sqrt{n}}&=&lim_{mto +infty}sum_{ngeq 1}frac{1}{sqrt{n}}int_{0}^{+infty}msin(nx) e^{-mx},dx\&=&lim_{mto +infty}sum_{ngeq 1}frac{1}{sqrt{n}}cdotfrac{mn}{m^2+n^2}.end{eqnarray*}$$
                          The RHS is $+infty$ since by Riemann sums
                          $$ lim_{mto +infty}sum_{ngeq 1}frac{sqrt{mn}}{m^2+n^2}=int_{0}^{+infty}frac{sqrt{x}}{1+x^2},dx = frac{pi}{sqrt{2}}.$$
                          Much simpler, once $f(x)$ is defined as $sum_{ngeq 1}frac{sin(nx)}{sqrt{n}}$ we have
                          $$ int_{-pi}^{pi}f(x)^2,dx = pisum_{ngeq 1}frac{1}{n} = +infty $$
                          by Parseval's theorem, hence $f(x)$ cannot be bounded on $[-pi,pi]$.






                          share|cite|improve this answer














                          Pointwise convergence is granted by Dirichlet's test, but a series of continuous functions cannot converge uniformly to an unbounded function. We may compute $lim_{xto 0^+}sum_{ngeq 1}frac{sin(n x)}{sqrt{n}}$ through a convolution with an approximate identity:



                          $$begin{eqnarray*}lim_{xto 0^+}sum_{ngeq 1}frac{sin(n x)}{sqrt{n}}&=&lim_{mto +infty}sum_{ngeq 1}frac{1}{sqrt{n}}int_{0}^{+infty}msin(nx) e^{-mx},dx\&=&lim_{mto +infty}sum_{ngeq 1}frac{1}{sqrt{n}}cdotfrac{mn}{m^2+n^2}.end{eqnarray*}$$
                          The RHS is $+infty$ since by Riemann sums
                          $$ lim_{mto +infty}sum_{ngeq 1}frac{sqrt{mn}}{m^2+n^2}=int_{0}^{+infty}frac{sqrt{x}}{1+x^2},dx = frac{pi}{sqrt{2}}.$$
                          Much simpler, once $f(x)$ is defined as $sum_{ngeq 1}frac{sin(nx)}{sqrt{n}}$ we have
                          $$ int_{-pi}^{pi}f(x)^2,dx = pisum_{ngeq 1}frac{1}{n} = +infty $$
                          by Parseval's theorem, hence $f(x)$ cannot be bounded on $[-pi,pi]$.







                          share|cite|improve this answer














                          share|cite|improve this answer



                          share|cite|improve this answer








                          edited Nov 21 at 21:49

























                          answered Nov 21 at 21:34









                          Jack D'Aurizio

                          284k33275654




                          284k33275654






























                              draft saved

                              draft discarded




















































                              Thanks for contributing an answer to Mathematics Stack Exchange!


                              • Please be sure to answer the question. Provide details and share your research!

                              But avoid



                              • Asking for help, clarification, or responding to other answers.

                              • Making statements based on opinion; back them up with references or personal experience.


                              Use MathJax to format equations. MathJax reference.


                              To learn more, see our tips on writing great answers.





                              Some of your past answers have not been well-received, and you're in danger of being blocked from answering.


                              Please pay close attention to the following guidance:


                              • Please be sure to answer the question. Provide details and share your research!

                              But avoid



                              • Asking for help, clarification, or responding to other answers.

                              • Making statements based on opinion; back them up with references or personal experience.


                              To learn more, see our tips on writing great answers.




                              draft saved


                              draft discarded














                              StackExchange.ready(
                              function () {
                              StackExchange.openid.initPostLogin('.new-post-login', 'https%3a%2f%2fmath.stackexchange.com%2fquestions%2f3008359%2fconverge-pointwise-but-not-uniformly%23new-answer', 'question_page');
                              }
                              );

                              Post as a guest















                              Required, but never shown





















































                              Required, but never shown














                              Required, but never shown












                              Required, but never shown







                              Required, but never shown

































                              Required, but never shown














                              Required, but never shown












                              Required, but never shown







                              Required, but never shown







                              Popular posts from this blog

                              Le Mesnil-Réaume

                              Ida-Boy-Ed-Garten

                              web3.py web3.isConnected() returns false always